LSAT and Law School Admissions Forum

Get expert LSAT preparation and law school admissions advice from PowerScore Test Preparation.

 Administrator
PowerScore Staff
  • PowerScore Staff
  • Posts: 8916
  • Joined: Feb 02, 2011
|
#24826
Complete Question Explanation

Assumption. The correct answer choice is (B)

The conclusion of this stimulus is presented in the first sentence, and followed by the supporting premises. The argument is basically as follows:
  • Premise: ..... Computers can only solve problems with mechanically applicable rules.

    Premise: ..... Some problems can not be solved with mechanically applicable rules.

    Conclusion: ..... Therefore computers will never be able to do what some human minds do.
The author must believe that some human minds can solve problems that cannot be solved with mechanically applicable rules. Only correct answer choice (B) provides this assumption: the author must believe that at least one problem that cannot be solved mechanically can be solved by some humans (recall that the word “some” is equivalent to “at least one”).
 PeterC123
  • Posts: 26
  • Joined: Dec 27, 2016
|
#31619
Hi,

I understood this question through intuition, but when I was reviewing this question I had trouble getting rid of A, C, D, and E. I tried to draw the conditional statement for the argument and tried to link the elements up and could not do it.


Can you show me how to disprove the rest of the answers by using conditional chain or another method that you can use to disprove them.

I know sometimes you should not approach certain questions by drawing out the conditional chains, is this one of them? And should you draw conditional chains when you can easily see what element links with what element?

Thanks,
 Kristina Moen
PowerScore Staff
  • PowerScore Staff
  • Posts: 230
  • Joined: Nov 17, 2016
|
#31664
Hi Peter,

Here is how I would understand the conditional reasoning in this argument.

Premise 1: Computer solve problems -> following some set of mechanically applicable rules
Premise 2: There exists some problems that cannot be solved by following some set of mechanically applicable rules.

Use the contrapositive of Premise 1, which would lead to the conclusion that there are some problems that cannot be solved by computers. That would be a logical argument.

Instead, the argument concludes that there exists some problems that can be solved by a human mind, but not computers ("No computer will ever be able to do everything that some human minds can do").

When approaching this stimulus, I note the "new" (or "rogue") element in the conclusion. The "new" element is the human mind. While this is an Assumption question, not a Justify question, it is helpful to note the "new" element in any argument. I especially appreciate "new" elements in arguments that contain conditional reasoning, as it is often much easier to see where the argument went off course.

Peter, try using the Assumption Negation Technique with the rest of the answer choices. Keep in mind that the logical opposite of "At least one" is "None" and the logical opposite of "Every" is "Not Every." You want to negate the main subject. I'll show you Answer Choice (E) and you can come back and explain why the other answer choices are wrong. One of the best ways to learn is to explain it to someone else!

Answer Choice (E) would be negated to "Not every problem that is solvable by following at least one set of mechanically applicable rules is solvable by at least one human mind" i.e. there exists some problem that meets the necessary condition for computers, but is not solvable by any human. Well, that doesn't kill the conclusion. The conclusion is not that humans can solve all the same problems that computers can solve, but that there exists at least one problem that a human mind can solve, but computers cannot solve. Furthermore, just because a problem meets the necessary condition for a computer does not mean it's actually solvable by a computer! So maybe there's a problem that is not solvable by a computer OR a human. That still doesn't kill the conclusion.

I look forward to reading your approach to the other answer choices!
 PeterC123
  • Posts: 26
  • Joined: Dec 27, 2016
|
#31687
Hi Kristina,

The conclusion is saying that computers are not able to do everything human minds can do, so one of the answer choices must have something that says human minds can do something that the computer can't.

Negated version of A: No problem solvable by following some set of mechanically applicable rules is not solvable by any human mind. Meaning every problem solvable by following some rules is solvable by human mind. This would not kill the conclusion b/c this just means that human minds and computers are equally able to solve the problems by following rules, but it says nothing about what human minds can do with problems that cannot be solved using mechanically applicable rules, which is what the conclusion banks on.

Negated version of C: No problem solvable by following some set of mechanically applicable rules is solvable by every human mind. Meaning human minds cannot do the things computers do, but it does not kill the conclusion b/c even though humans can't do what the computers can do, maybe computers cannot do what human minds can do (the answer choice does not touch on this), therefore the conclusion still works.

Negated version of D: Not every problem that is solvable by following more than one set of mechanically applicable rules is solvable by almost every human mind. Again, we don't really care whether human minds can do what computers do, all we want to know is can human minds solve problems that computers cannot. This answer choice therefore does not kill the conclusion.

I can see now that every other answer choice beside B talks about whether or not human minds can solve the problems computer can solve, which is not the point of the conclusion. This has been surprisingly helpful!

Thanks,
 Kristina Moen
PowerScore Staff
  • PowerScore Staff
  • Posts: 230
  • Joined: Nov 17, 2016
|
#31724
Peter, nice job! Keep it up! :)

Get the most out of your LSAT Prep Plus subscription.

Analyze and track your performance with our Testing and Analytics Package.